LSAT and Law School Admissions Forum

Get expert LSAT preparation and law school admissions advice from PowerScore Test Preparation.

 James Finch
PowerScore Staff
  • PowerScore Staff
  • Posts: 943
  • Joined: Sep 06, 2017
|
#65396
Hi Oli,

Young is effectively calling out West for a numbers and percentages flaw. West is assuming, based only on a single raw number, that Haynes has a worse defect rate than the other two inspectors. However, as Young points out, this isn't the case, because Young inspects more than half of the products, so only having inspected half of the defective returns means that his defective rate is actually lower than the other two combined (although one of the two could have a lower defect rate, at least one of the other inspectors must be higher. So that's the Prephrase, and then we have to match an answer choice to it.

(A) is talking about circular reasoning, which is neither present nor what Young is accusing West of. Incorrect.

(B) is talking about "relevance" of a conclusion, which doesn't sound like a flaw and is definitely not what Young is doing. Incorrect.

(C) is interesting, but isn't actually what Young is doing; Young is quibbling with an assumption being made by West, not a stated premise, so this is incorrect.

(D) is clearly incorrect, as Young is attacking West's assumption, not arguing for a similar conclusion.

(E) is talking about a "presupposition" which here is synonymous with an assumption. This is what Young is doing, as he is attacking the assumption that because more than half of the raw number of defective returns were inspected by Haynes, Haynes's defective rate must be higher than that of the other two inspectors. However, one number alone isn't enough to give us a percentage, and Young's second number tells us that the rate would actually be lower than the other two. So (E) is correct, in that it accurately describes Young's attack on West's assumption.

Hope this clears things up!
 ShannonOh22
  • Posts: 70
  • Joined: Aug 15, 2019
|
#68509
Hi PowerScore,

I was between C and E on this question, and I ended up going with C for the following reasons:

By Young saying "But Haynes inspects significantly more than half the appliances we sell each year", isn't that disputing the accuracy of West's stated premise that "Of our company's three quality control inspectors, Haynes is clearly the worst."? The point of their dialogue is that they disagree on Haynes' being considered the "worst" inspector, right?

E looked appealing (obviously, seeing as it is correct :), but I opted for C because E seems too vague. I am always wary of going with an answer choice that includes "presupposition", because it is much harder to defend. We don't actually KNOW what West was presupposing...maybe the fact that half of the products that were returned was due to some other factor that Haynes had a part in...maybe he has poor eyesight, or he shows up late, or he doesn't agree with one of the "quality control standards" the company has in place...whatever it is, we can't KNOW for sure what his presupposition is, so how do we choose that answer choice with confidence?

Lastly, the wording of the answer choices contributed to my decision to choose C. E is too vague...nothing in Young's response includes an actual denial (which would be expressed by saying "no", or "not necessarily", or "that isn't true because" etc.)...but it DOES include a dispute - "But".

Please let me know how to become savvier on these types of questions...and thank you in advance!!
 James Finch
PowerScore Staff
  • PowerScore Staff
  • Posts: 943
  • Joined: Sep 06, 2017
|
#70981
Hi Shannon,

No, Young is disputing West's conclusion and the validity of it based on the premise that Haynes inspected half of the defective products. Young accepts West's premise while attacking his conclusion by adding another premise, a counter-premise, that is necessary to actually draw any inferences based on proportions: a second proportion that tells us the rough proportion of the total number of products Haynes inspected, allowing us to get a rough idea of Haynes's defect rate. Based on this new information, Haynes must actually be a better inspector than the other two in aggregate (although one could be terrible and the other even better than Haynes, so we can't say Haynes is necessarily the best) meaning that Haynes can't be the worst inspector.

West seems to be operating under the assumption that all threes inspectors inspected the same amount of total products, and thus that each should also account for roughly 1/3rd of actually defective products. This would also mean that if any of them had inspected more than 1/3rd of the defective products, they would be worse at their job than the other two inspectors. But as Young points out, this isn't necessarily true, and each inspector could have inspected a disproportionate amount of the total products, which turns out to be the case. By eliminating West's assumption, Young completely invalidates his conclusion as well. Answer choice (E) represents this ("presupposition" being synonymous with "assumption"), making it correct.

Hope this clears things up!
User avatar
 queenbee
  • Posts: 75
  • Joined: Sep 18, 2022
|
#98077
Hi
I thought that C was the best answer because West was misinterpreting the data he was observing.
I also didnt see Young denying West's claim. If Young had said, "you are wrong," or something like that I would have have given E more weight. But Young provided another data point for West to evaluate when considering the number of units being returned. Am I just reading this wrong?

Thank you
User avatar
 Paul Popa
PowerScore Staff
  • PowerScore Staff
  • Posts: 64
  • Joined: Sep 20, 2022
|
#98500
Hey Queen,

Great question! There's a difference between misinterpreting data, and stating that the data is inaccurate. Young didn't actually contradict a premise; he accepts the data as is, but adds additional information to better show the whole story. An example of (C) is if Young had said "Actually, it's not true that half of the items returned had been inspected by Haynes, it was actually closer to 25%."

When considering (E), let's first determine if there is indeed a presupposition, or unstated premise, in West's argument. I see one: West is assuming that each of the inspectors inspected equal (or at least similar) amounts of appliances; that's why the 50% rate appears so damning. But if, as Young is arguing, that Haynes inspects a significant majority of the appliances, the 50% rate is actually not so bad. Hope this helps!

Get the most out of your LSAT Prep Plus subscription.

Analyze and track your performance with our Testing and Analytics Package.